Determining the missing digits of $15! equiv 1square0767436square000$ without actually calculating the...












6












$begingroup$



$$15! equiv 1cdot 2cdot 3cdot,cdots,cdot 15 equiv 1square0767436square000$$



Using a calculator, I know that the answer is $3$ and $8$, but I know that the answer can be calculated by hand.




How to calculate the missing digits? I know that large factorials can be estimated using Stirling's approximation:
$$15! approx sqrt{2picdot 15}
cdot left(frac{15}{e}right)^{15}$$

which is not feasible to calculate by hand.



The resulting number must be divisible by 9 which means that the digit sum must add up to 9 and is also divisible by 11 which means that the alternating digit sum must be divisible by 11:



$1+ d_0 + 0 + 7 +6 +7 +4 +3+6+d_1+0+0+0 mod phantom{1}9 equiv ,34 + d_0 + d_1 mod phantom{1}9 equiv 0 $
$-1+ d_0 - 0 + 7 -6 +7 -4 +3-6+d_1-0+0-0 mod 11 equiv d_0 + d_1 mod 11 equiv 0 $



The digits $3$ and $8$, or $7$ and $4$, fulfill both of the requirements.










share|cite|improve this question











$endgroup$








  • 8




    $begingroup$
    Well, you know that the sum of the digits is divisible by $9$ and the alternating sum of the digits is divisible by $11$. That should tell you what the two digits are, though not their order. But you should be able to argue that the first non-zero digit from the right must be even.
    $endgroup$
    – lulu
    Jan 2 at 21:36










  • $begingroup$
    Why is the sum of the digits divisible by 9?
    $endgroup$
    – Darkice
    Jan 2 at 21:37






  • 3




    $begingroup$
    @Darkice because for every natural number $ngeq 6$ you have that $n!$ is a multiple of nine and a natural number is a multiple of nine if and only if the sum of its digits is a multiple of nine.
    $endgroup$
    – JMoravitz
    Jan 2 at 21:41








  • 1




    $begingroup$
    Alright, now I know that the digits must add up to a multiple of 9 and must be divisible by 11. So the digits could be 7 and 4 or 3 and 8 which would both fulfil the requirements.
    $endgroup$
    – Darkice
    Jan 2 at 21:59






  • 6




    $begingroup$
    For your specific case, one of the numbers you are looking to find is the final nonzero digit. As $15! = 2^{11}cdot3^6cdot5^3cdot7^2cdot11cdot13$ the final nonzero digit will be $2^8cdot3^6cdot7^2cdot11cdot13pmod{10}$ which is a much easier calculation to do modulo 10 than fully expanding out the product. $2^8cdot3^6cdot7^2cdot11cdot13equiv 6cdot 9cdot 9cdot 1cdot 3equiv 8pmod{10}$. This would be admittedly more difficult if both of the numbers were located further in the middle and not next to the trailing zeroes.
    $endgroup$
    – JMoravitz
    Jan 2 at 22:09


















6












$begingroup$



$$15! equiv 1cdot 2cdot 3cdot,cdots,cdot 15 equiv 1square0767436square000$$



Using a calculator, I know that the answer is $3$ and $8$, but I know that the answer can be calculated by hand.




How to calculate the missing digits? I know that large factorials can be estimated using Stirling's approximation:
$$15! approx sqrt{2picdot 15}
cdot left(frac{15}{e}right)^{15}$$

which is not feasible to calculate by hand.



The resulting number must be divisible by 9 which means that the digit sum must add up to 9 and is also divisible by 11 which means that the alternating digit sum must be divisible by 11:



$1+ d_0 + 0 + 7 +6 +7 +4 +3+6+d_1+0+0+0 mod phantom{1}9 equiv ,34 + d_0 + d_1 mod phantom{1}9 equiv 0 $
$-1+ d_0 - 0 + 7 -6 +7 -4 +3-6+d_1-0+0-0 mod 11 equiv d_0 + d_1 mod 11 equiv 0 $



The digits $3$ and $8$, or $7$ and $4$, fulfill both of the requirements.










share|cite|improve this question











$endgroup$








  • 8




    $begingroup$
    Well, you know that the sum of the digits is divisible by $9$ and the alternating sum of the digits is divisible by $11$. That should tell you what the two digits are, though not their order. But you should be able to argue that the first non-zero digit from the right must be even.
    $endgroup$
    – lulu
    Jan 2 at 21:36










  • $begingroup$
    Why is the sum of the digits divisible by 9?
    $endgroup$
    – Darkice
    Jan 2 at 21:37






  • 3




    $begingroup$
    @Darkice because for every natural number $ngeq 6$ you have that $n!$ is a multiple of nine and a natural number is a multiple of nine if and only if the sum of its digits is a multiple of nine.
    $endgroup$
    – JMoravitz
    Jan 2 at 21:41








  • 1




    $begingroup$
    Alright, now I know that the digits must add up to a multiple of 9 and must be divisible by 11. So the digits could be 7 and 4 or 3 and 8 which would both fulfil the requirements.
    $endgroup$
    – Darkice
    Jan 2 at 21:59






  • 6




    $begingroup$
    For your specific case, one of the numbers you are looking to find is the final nonzero digit. As $15! = 2^{11}cdot3^6cdot5^3cdot7^2cdot11cdot13$ the final nonzero digit will be $2^8cdot3^6cdot7^2cdot11cdot13pmod{10}$ which is a much easier calculation to do modulo 10 than fully expanding out the product. $2^8cdot3^6cdot7^2cdot11cdot13equiv 6cdot 9cdot 9cdot 1cdot 3equiv 8pmod{10}$. This would be admittedly more difficult if both of the numbers were located further in the middle and not next to the trailing zeroes.
    $endgroup$
    – JMoravitz
    Jan 2 at 22:09
















6












6








6


1



$begingroup$



$$15! equiv 1cdot 2cdot 3cdot,cdots,cdot 15 equiv 1square0767436square000$$



Using a calculator, I know that the answer is $3$ and $8$, but I know that the answer can be calculated by hand.




How to calculate the missing digits? I know that large factorials can be estimated using Stirling's approximation:
$$15! approx sqrt{2picdot 15}
cdot left(frac{15}{e}right)^{15}$$

which is not feasible to calculate by hand.



The resulting number must be divisible by 9 which means that the digit sum must add up to 9 and is also divisible by 11 which means that the alternating digit sum must be divisible by 11:



$1+ d_0 + 0 + 7 +6 +7 +4 +3+6+d_1+0+0+0 mod phantom{1}9 equiv ,34 + d_0 + d_1 mod phantom{1}9 equiv 0 $
$-1+ d_0 - 0 + 7 -6 +7 -4 +3-6+d_1-0+0-0 mod 11 equiv d_0 + d_1 mod 11 equiv 0 $



The digits $3$ and $8$, or $7$ and $4$, fulfill both of the requirements.










share|cite|improve this question











$endgroup$





$$15! equiv 1cdot 2cdot 3cdot,cdots,cdot 15 equiv 1square0767436square000$$



Using a calculator, I know that the answer is $3$ and $8$, but I know that the answer can be calculated by hand.




How to calculate the missing digits? I know that large factorials can be estimated using Stirling's approximation:
$$15! approx sqrt{2picdot 15}
cdot left(frac{15}{e}right)^{15}$$

which is not feasible to calculate by hand.



The resulting number must be divisible by 9 which means that the digit sum must add up to 9 and is also divisible by 11 which means that the alternating digit sum must be divisible by 11:



$1+ d_0 + 0 + 7 +6 +7 +4 +3+6+d_1+0+0+0 mod phantom{1}9 equiv ,34 + d_0 + d_1 mod phantom{1}9 equiv 0 $
$-1+ d_0 - 0 + 7 -6 +7 -4 +3-6+d_1-0+0-0 mod 11 equiv d_0 + d_1 mod 11 equiv 0 $



The digits $3$ and $8$, or $7$ and $4$, fulfill both of the requirements.







elementary-number-theory factorial






share|cite|improve this question















share|cite|improve this question













share|cite|improve this question




share|cite|improve this question








edited Jan 2 at 23:48







user630758

















asked Jan 2 at 21:30









DarkiceDarkice

1415




1415








  • 8




    $begingroup$
    Well, you know that the sum of the digits is divisible by $9$ and the alternating sum of the digits is divisible by $11$. That should tell you what the two digits are, though not their order. But you should be able to argue that the first non-zero digit from the right must be even.
    $endgroup$
    – lulu
    Jan 2 at 21:36










  • $begingroup$
    Why is the sum of the digits divisible by 9?
    $endgroup$
    – Darkice
    Jan 2 at 21:37






  • 3




    $begingroup$
    @Darkice because for every natural number $ngeq 6$ you have that $n!$ is a multiple of nine and a natural number is a multiple of nine if and only if the sum of its digits is a multiple of nine.
    $endgroup$
    – JMoravitz
    Jan 2 at 21:41








  • 1




    $begingroup$
    Alright, now I know that the digits must add up to a multiple of 9 and must be divisible by 11. So the digits could be 7 and 4 or 3 and 8 which would both fulfil the requirements.
    $endgroup$
    – Darkice
    Jan 2 at 21:59






  • 6




    $begingroup$
    For your specific case, one of the numbers you are looking to find is the final nonzero digit. As $15! = 2^{11}cdot3^6cdot5^3cdot7^2cdot11cdot13$ the final nonzero digit will be $2^8cdot3^6cdot7^2cdot11cdot13pmod{10}$ which is a much easier calculation to do modulo 10 than fully expanding out the product. $2^8cdot3^6cdot7^2cdot11cdot13equiv 6cdot 9cdot 9cdot 1cdot 3equiv 8pmod{10}$. This would be admittedly more difficult if both of the numbers were located further in the middle and not next to the trailing zeroes.
    $endgroup$
    – JMoravitz
    Jan 2 at 22:09
















  • 8




    $begingroup$
    Well, you know that the sum of the digits is divisible by $9$ and the alternating sum of the digits is divisible by $11$. That should tell you what the two digits are, though not their order. But you should be able to argue that the first non-zero digit from the right must be even.
    $endgroup$
    – lulu
    Jan 2 at 21:36










  • $begingroup$
    Why is the sum of the digits divisible by 9?
    $endgroup$
    – Darkice
    Jan 2 at 21:37






  • 3




    $begingroup$
    @Darkice because for every natural number $ngeq 6$ you have that $n!$ is a multiple of nine and a natural number is a multiple of nine if and only if the sum of its digits is a multiple of nine.
    $endgroup$
    – JMoravitz
    Jan 2 at 21:41








  • 1




    $begingroup$
    Alright, now I know that the digits must add up to a multiple of 9 and must be divisible by 11. So the digits could be 7 and 4 or 3 and 8 which would both fulfil the requirements.
    $endgroup$
    – Darkice
    Jan 2 at 21:59






  • 6




    $begingroup$
    For your specific case, one of the numbers you are looking to find is the final nonzero digit. As $15! = 2^{11}cdot3^6cdot5^3cdot7^2cdot11cdot13$ the final nonzero digit will be $2^8cdot3^6cdot7^2cdot11cdot13pmod{10}$ which is a much easier calculation to do modulo 10 than fully expanding out the product. $2^8cdot3^6cdot7^2cdot11cdot13equiv 6cdot 9cdot 9cdot 1cdot 3equiv 8pmod{10}$. This would be admittedly more difficult if both of the numbers were located further in the middle and not next to the trailing zeroes.
    $endgroup$
    – JMoravitz
    Jan 2 at 22:09










8




8




$begingroup$
Well, you know that the sum of the digits is divisible by $9$ and the alternating sum of the digits is divisible by $11$. That should tell you what the two digits are, though not their order. But you should be able to argue that the first non-zero digit from the right must be even.
$endgroup$
– lulu
Jan 2 at 21:36




$begingroup$
Well, you know that the sum of the digits is divisible by $9$ and the alternating sum of the digits is divisible by $11$. That should tell you what the two digits are, though not their order. But you should be able to argue that the first non-zero digit from the right must be even.
$endgroup$
– lulu
Jan 2 at 21:36












$begingroup$
Why is the sum of the digits divisible by 9?
$endgroup$
– Darkice
Jan 2 at 21:37




$begingroup$
Why is the sum of the digits divisible by 9?
$endgroup$
– Darkice
Jan 2 at 21:37




3




3




$begingroup$
@Darkice because for every natural number $ngeq 6$ you have that $n!$ is a multiple of nine and a natural number is a multiple of nine if and only if the sum of its digits is a multiple of nine.
$endgroup$
– JMoravitz
Jan 2 at 21:41






$begingroup$
@Darkice because for every natural number $ngeq 6$ you have that $n!$ is a multiple of nine and a natural number is a multiple of nine if and only if the sum of its digits is a multiple of nine.
$endgroup$
– JMoravitz
Jan 2 at 21:41






1




1




$begingroup$
Alright, now I know that the digits must add up to a multiple of 9 and must be divisible by 11. So the digits could be 7 and 4 or 3 and 8 which would both fulfil the requirements.
$endgroup$
– Darkice
Jan 2 at 21:59




$begingroup$
Alright, now I know that the digits must add up to a multiple of 9 and must be divisible by 11. So the digits could be 7 and 4 or 3 and 8 which would both fulfil the requirements.
$endgroup$
– Darkice
Jan 2 at 21:59




6




6




$begingroup$
For your specific case, one of the numbers you are looking to find is the final nonzero digit. As $15! = 2^{11}cdot3^6cdot5^3cdot7^2cdot11cdot13$ the final nonzero digit will be $2^8cdot3^6cdot7^2cdot11cdot13pmod{10}$ which is a much easier calculation to do modulo 10 than fully expanding out the product. $2^8cdot3^6cdot7^2cdot11cdot13equiv 6cdot 9cdot 9cdot 1cdot 3equiv 8pmod{10}$. This would be admittedly more difficult if both of the numbers were located further in the middle and not next to the trailing zeroes.
$endgroup$
– JMoravitz
Jan 2 at 22:09






$begingroup$
For your specific case, one of the numbers you are looking to find is the final nonzero digit. As $15! = 2^{11}cdot3^6cdot5^3cdot7^2cdot11cdot13$ the final nonzero digit will be $2^8cdot3^6cdot7^2cdot11cdot13pmod{10}$ which is a much easier calculation to do modulo 10 than fully expanding out the product. $2^8cdot3^6cdot7^2cdot11cdot13equiv 6cdot 9cdot 9cdot 1cdot 3equiv 8pmod{10}$. This would be admittedly more difficult if both of the numbers were located further in the middle and not next to the trailing zeroes.
$endgroup$
– JMoravitz
Jan 2 at 22:09












6 Answers
6






active

oldest

votes


















3












$begingroup$

Another way to reason is to note that $15!$ is divisible by $2cdot4cdot2cdot8cdot2cdot4cdot2=2^{11}$, which means $1square0767436square$ is divisible by $2^8$. In particular, it's divisible by $8$. But since $8mid1000$ and $8mid360$, the final $square$ must be either $0$ or $8$. But it can't be $0$, since $15!$ has only three powers of $5$ (from $5$, $10$, and $15$), and those were already accounted for in the final three $0$'s of the number. Thus the final $square$ is an $8$. Casting out $9$'s now reveals that the first $square$ is a $3$.



Remark: It wasn't strictly necessary to determine the exact power of $2$ (namely $2^{11}$) that divides $15!$, merely that $2^6$ divides it, but it wasn't that hard to do.






share|cite|improve this answer









$endgroup$





















    2












    $begingroup$

    You can cast out $9$’s and $11$’s:
    begin{align}
    1+x+0+7+6+7+4+3+6+y+0+0+0&=x+y+34 \
    1-x+0-7+6-7+4-3+6-y+0-0+0&=-x-y
    end{align}

    Thus $x+y=11$ (it can't be $x=y=0$).



    Then find the remainder modulo $10000$; since
    $$
    15!=2^{11}cdot 3^6cdot 5^3cdot 7^2cdot11cdot13=1000cdot 2^8cdot3^6cdot7^2cdot 11cdot 13
    $$

    this means finding the remainder modulo $10$ of
    $$
    2^8cdot3^6cdot7^2cdot 11cdot 13
    $$

    that gives $8$ with a short computation.






    share|cite|improve this answer









    $endgroup$





















      1












      $begingroup$

      Using the divisibility rule for 7 the answer boils down to 3 and 8:



      $-368+674+307+1 mod 7 equiv 0$






      share|cite|improve this answer









      $endgroup$













      • $begingroup$
        for the order problem you could remove three zeros from the Least significant digit side and say it still needs to be divisible by 2 or 4 cause there are lots of 2's and 4's in the original multiplication, so 3 couldn't be on the right side.
        $endgroup$
        – no0ob
        Jan 2 at 23:34












      • $begingroup$
        The order is also given by the divisibilty rule for 7.
        $endgroup$
        – Darkice
        Jan 3 at 8:54



















      1












      $begingroup$

      Okay, $15! = 1*2*3..... *15=1a0767436b000$.



      Why does it end with $000$? Well, obviously because $5,10,15$ all divide into it so $5^3$ divides into it and at least three copies of $2$ divid into it so $2^3*5^3 =1000$ divide into it.



      If we divide $15!$ by $100 = 8*5^3$ we get



      $1a0767436b = 1*2*3*4*6*7*9*2*11*12*13*14*3$



      If we want to find the last digit of that we can do



      $1a0767436b equiv b pmod {10}$ and



      $1*2*3*4*6*7*9*2*11*12*13*14*3equiv 2*3*4*(-4)*(-3)*(-1)*2*1*2*3*4*3equiv$



      $-2^9*3^4 equiv -512*81equiv -2 equiv 8pmod {10}$..



      So $b = 8$.



      But what is $a$?



      Well, $11|1a0767436b$ and $9|1a0767436b$.



      So $1+0+6+4+6 - a - 7-7-3-b = 11k$ for some integer $k$. And $1+a+0+7+6+7+4+3+6+b = 9j$ for some integer $j$.



      So $-a -8 =11k$ so as $0le a le 9$ we have $a = 3$.



      And that's that $15! = 1307674368000$..... IF we assume the person who asked this question was telling the truth.



      We do know that $15!$ ends with $.... 8000$ but we are completely taking someone elses word for it that it begins with $1a0767436....$






      share|cite|improve this answer









      $endgroup$





















        1












        $begingroup$

        Let $d_1$ and $d_2$ be the two unknown digits.



        The number must be divisible by $8000$, because $15!$ contains $8$ and $1000$.



        $d_2$ is non-zero number, because $15!$ contains only three $5$s. It implies $1d_10767436d_2$ must be divisible by $8$. It implies $36d_2$ is divisible by $8$. Hence, $d_2=8$.



        Now you can use the divisibility by $9$ ($d_1+d_2=11$) and find $d_1=3$.






        share|cite|improve this answer









        $endgroup$





















          1












          $begingroup$

          $15!=2^{11}cdot 5^3cdot 7^2cdot 11cdot 13=(1000)X$ where $X=2^8cdot 3^6cdot 7^2cdot 11cdot 13.$



          The last digits of $2^8(=16^2), 3^6 (=9^3),7^2, 11,13$ are, respectively $6,9,9,1,3 .$



          Modulo $10$ we have $6cdot 9cdot 9 cdot 1cdot 3equiv 6cdot(-1)^2cdot 3equiv 18equiv 8$. So the last digit of $X$ is an $8$.



          Therefore the 2nd digit of $15!$ must be a $3$ in order for the sum of all its digits to be divisible by $9$.






          share|cite|improve this answer









          $endgroup$













            Your Answer





            StackExchange.ifUsing("editor", function () {
            return StackExchange.using("mathjaxEditing", function () {
            StackExchange.MarkdownEditor.creationCallbacks.add(function (editor, postfix) {
            StackExchange.mathjaxEditing.prepareWmdForMathJax(editor, postfix, [["$", "$"], ["\\(","\\)"]]);
            });
            });
            }, "mathjax-editing");

            StackExchange.ready(function() {
            var channelOptions = {
            tags: "".split(" "),
            id: "69"
            };
            initTagRenderer("".split(" "), "".split(" "), channelOptions);

            StackExchange.using("externalEditor", function() {
            // Have to fire editor after snippets, if snippets enabled
            if (StackExchange.settings.snippets.snippetsEnabled) {
            StackExchange.using("snippets", function() {
            createEditor();
            });
            }
            else {
            createEditor();
            }
            });

            function createEditor() {
            StackExchange.prepareEditor({
            heartbeatType: 'answer',
            autoActivateHeartbeat: false,
            convertImagesToLinks: true,
            noModals: true,
            showLowRepImageUploadWarning: true,
            reputationToPostImages: 10,
            bindNavPrevention: true,
            postfix: "",
            imageUploader: {
            brandingHtml: "Powered by u003ca class="icon-imgur-white" href="https://imgur.com/"u003eu003c/au003e",
            contentPolicyHtml: "User contributions licensed under u003ca href="https://creativecommons.org/licenses/by-sa/3.0/"u003ecc by-sa 3.0 with attribution requiredu003c/au003e u003ca href="https://stackoverflow.com/legal/content-policy"u003e(content policy)u003c/au003e",
            allowUrls: true
            },
            noCode: true, onDemand: true,
            discardSelector: ".discard-answer"
            ,immediatelyShowMarkdownHelp:true
            });


            }
            });














            draft saved

            draft discarded


















            StackExchange.ready(
            function () {
            StackExchange.openid.initPostLogin('.new-post-login', 'https%3a%2f%2fmath.stackexchange.com%2fquestions%2f3059994%2fdetermining-the-missing-digits-of-15-equiv-1-square0767436-square000-without%23new-answer', 'question_page');
            }
            );

            Post as a guest















            Required, but never shown

























            6 Answers
            6






            active

            oldest

            votes








            6 Answers
            6






            active

            oldest

            votes









            active

            oldest

            votes






            active

            oldest

            votes









            3












            $begingroup$

            Another way to reason is to note that $15!$ is divisible by $2cdot4cdot2cdot8cdot2cdot4cdot2=2^{11}$, which means $1square0767436square$ is divisible by $2^8$. In particular, it's divisible by $8$. But since $8mid1000$ and $8mid360$, the final $square$ must be either $0$ or $8$. But it can't be $0$, since $15!$ has only three powers of $5$ (from $5$, $10$, and $15$), and those were already accounted for in the final three $0$'s of the number. Thus the final $square$ is an $8$. Casting out $9$'s now reveals that the first $square$ is a $3$.



            Remark: It wasn't strictly necessary to determine the exact power of $2$ (namely $2^{11}$) that divides $15!$, merely that $2^6$ divides it, but it wasn't that hard to do.






            share|cite|improve this answer









            $endgroup$


















              3












              $begingroup$

              Another way to reason is to note that $15!$ is divisible by $2cdot4cdot2cdot8cdot2cdot4cdot2=2^{11}$, which means $1square0767436square$ is divisible by $2^8$. In particular, it's divisible by $8$. But since $8mid1000$ and $8mid360$, the final $square$ must be either $0$ or $8$. But it can't be $0$, since $15!$ has only three powers of $5$ (from $5$, $10$, and $15$), and those were already accounted for in the final three $0$'s of the number. Thus the final $square$ is an $8$. Casting out $9$'s now reveals that the first $square$ is a $3$.



              Remark: It wasn't strictly necessary to determine the exact power of $2$ (namely $2^{11}$) that divides $15!$, merely that $2^6$ divides it, but it wasn't that hard to do.






              share|cite|improve this answer









              $endgroup$
















                3












                3








                3





                $begingroup$

                Another way to reason is to note that $15!$ is divisible by $2cdot4cdot2cdot8cdot2cdot4cdot2=2^{11}$, which means $1square0767436square$ is divisible by $2^8$. In particular, it's divisible by $8$. But since $8mid1000$ and $8mid360$, the final $square$ must be either $0$ or $8$. But it can't be $0$, since $15!$ has only three powers of $5$ (from $5$, $10$, and $15$), and those were already accounted for in the final three $0$'s of the number. Thus the final $square$ is an $8$. Casting out $9$'s now reveals that the first $square$ is a $3$.



                Remark: It wasn't strictly necessary to determine the exact power of $2$ (namely $2^{11}$) that divides $15!$, merely that $2^6$ divides it, but it wasn't that hard to do.






                share|cite|improve this answer









                $endgroup$



                Another way to reason is to note that $15!$ is divisible by $2cdot4cdot2cdot8cdot2cdot4cdot2=2^{11}$, which means $1square0767436square$ is divisible by $2^8$. In particular, it's divisible by $8$. But since $8mid1000$ and $8mid360$, the final $square$ must be either $0$ or $8$. But it can't be $0$, since $15!$ has only three powers of $5$ (from $5$, $10$, and $15$), and those were already accounted for in the final three $0$'s of the number. Thus the final $square$ is an $8$. Casting out $9$'s now reveals that the first $square$ is a $3$.



                Remark: It wasn't strictly necessary to determine the exact power of $2$ (namely $2^{11}$) that divides $15!$, merely that $2^6$ divides it, but it wasn't that hard to do.







                share|cite|improve this answer












                share|cite|improve this answer



                share|cite|improve this answer










                answered Jan 2 at 23:47









                Barry CipraBarry Cipra

                59.8k653126




                59.8k653126























                    2












                    $begingroup$

                    You can cast out $9$’s and $11$’s:
                    begin{align}
                    1+x+0+7+6+7+4+3+6+y+0+0+0&=x+y+34 \
                    1-x+0-7+6-7+4-3+6-y+0-0+0&=-x-y
                    end{align}

                    Thus $x+y=11$ (it can't be $x=y=0$).



                    Then find the remainder modulo $10000$; since
                    $$
                    15!=2^{11}cdot 3^6cdot 5^3cdot 7^2cdot11cdot13=1000cdot 2^8cdot3^6cdot7^2cdot 11cdot 13
                    $$

                    this means finding the remainder modulo $10$ of
                    $$
                    2^8cdot3^6cdot7^2cdot 11cdot 13
                    $$

                    that gives $8$ with a short computation.






                    share|cite|improve this answer









                    $endgroup$


















                      2












                      $begingroup$

                      You can cast out $9$’s and $11$’s:
                      begin{align}
                      1+x+0+7+6+7+4+3+6+y+0+0+0&=x+y+34 \
                      1-x+0-7+6-7+4-3+6-y+0-0+0&=-x-y
                      end{align}

                      Thus $x+y=11$ (it can't be $x=y=0$).



                      Then find the remainder modulo $10000$; since
                      $$
                      15!=2^{11}cdot 3^6cdot 5^3cdot 7^2cdot11cdot13=1000cdot 2^8cdot3^6cdot7^2cdot 11cdot 13
                      $$

                      this means finding the remainder modulo $10$ of
                      $$
                      2^8cdot3^6cdot7^2cdot 11cdot 13
                      $$

                      that gives $8$ with a short computation.






                      share|cite|improve this answer









                      $endgroup$
















                        2












                        2








                        2





                        $begingroup$

                        You can cast out $9$’s and $11$’s:
                        begin{align}
                        1+x+0+7+6+7+4+3+6+y+0+0+0&=x+y+34 \
                        1-x+0-7+6-7+4-3+6-y+0-0+0&=-x-y
                        end{align}

                        Thus $x+y=11$ (it can't be $x=y=0$).



                        Then find the remainder modulo $10000$; since
                        $$
                        15!=2^{11}cdot 3^6cdot 5^3cdot 7^2cdot11cdot13=1000cdot 2^8cdot3^6cdot7^2cdot 11cdot 13
                        $$

                        this means finding the remainder modulo $10$ of
                        $$
                        2^8cdot3^6cdot7^2cdot 11cdot 13
                        $$

                        that gives $8$ with a short computation.






                        share|cite|improve this answer









                        $endgroup$



                        You can cast out $9$’s and $11$’s:
                        begin{align}
                        1+x+0+7+6+7+4+3+6+y+0+0+0&=x+y+34 \
                        1-x+0-7+6-7+4-3+6-y+0-0+0&=-x-y
                        end{align}

                        Thus $x+y=11$ (it can't be $x=y=0$).



                        Then find the remainder modulo $10000$; since
                        $$
                        15!=2^{11}cdot 3^6cdot 5^3cdot 7^2cdot11cdot13=1000cdot 2^8cdot3^6cdot7^2cdot 11cdot 13
                        $$

                        this means finding the remainder modulo $10$ of
                        $$
                        2^8cdot3^6cdot7^2cdot 11cdot 13
                        $$

                        that gives $8$ with a short computation.







                        share|cite|improve this answer












                        share|cite|improve this answer



                        share|cite|improve this answer










                        answered Jan 2 at 23:49









                        egregegreg

                        183k1486205




                        183k1486205























                            1












                            $begingroup$

                            Using the divisibility rule for 7 the answer boils down to 3 and 8:



                            $-368+674+307+1 mod 7 equiv 0$






                            share|cite|improve this answer









                            $endgroup$













                            • $begingroup$
                              for the order problem you could remove three zeros from the Least significant digit side and say it still needs to be divisible by 2 or 4 cause there are lots of 2's and 4's in the original multiplication, so 3 couldn't be on the right side.
                              $endgroup$
                              – no0ob
                              Jan 2 at 23:34












                            • $begingroup$
                              The order is also given by the divisibilty rule for 7.
                              $endgroup$
                              – Darkice
                              Jan 3 at 8:54
















                            1












                            $begingroup$

                            Using the divisibility rule for 7 the answer boils down to 3 and 8:



                            $-368+674+307+1 mod 7 equiv 0$






                            share|cite|improve this answer









                            $endgroup$













                            • $begingroup$
                              for the order problem you could remove three zeros from the Least significant digit side and say it still needs to be divisible by 2 or 4 cause there are lots of 2's and 4's in the original multiplication, so 3 couldn't be on the right side.
                              $endgroup$
                              – no0ob
                              Jan 2 at 23:34












                            • $begingroup$
                              The order is also given by the divisibilty rule for 7.
                              $endgroup$
                              – Darkice
                              Jan 3 at 8:54














                            1












                            1








                            1





                            $begingroup$

                            Using the divisibility rule for 7 the answer boils down to 3 and 8:



                            $-368+674+307+1 mod 7 equiv 0$






                            share|cite|improve this answer









                            $endgroup$



                            Using the divisibility rule for 7 the answer boils down to 3 and 8:



                            $-368+674+307+1 mod 7 equiv 0$







                            share|cite|improve this answer












                            share|cite|improve this answer



                            share|cite|improve this answer










                            answered Jan 2 at 23:15









                            DarkiceDarkice

                            1415




                            1415












                            • $begingroup$
                              for the order problem you could remove three zeros from the Least significant digit side and say it still needs to be divisible by 2 or 4 cause there are lots of 2's and 4's in the original multiplication, so 3 couldn't be on the right side.
                              $endgroup$
                              – no0ob
                              Jan 2 at 23:34












                            • $begingroup$
                              The order is also given by the divisibilty rule for 7.
                              $endgroup$
                              – Darkice
                              Jan 3 at 8:54


















                            • $begingroup$
                              for the order problem you could remove three zeros from the Least significant digit side and say it still needs to be divisible by 2 or 4 cause there are lots of 2's and 4's in the original multiplication, so 3 couldn't be on the right side.
                              $endgroup$
                              – no0ob
                              Jan 2 at 23:34












                            • $begingroup$
                              The order is also given by the divisibilty rule for 7.
                              $endgroup$
                              – Darkice
                              Jan 3 at 8:54
















                            $begingroup$
                            for the order problem you could remove three zeros from the Least significant digit side and say it still needs to be divisible by 2 or 4 cause there are lots of 2's and 4's in the original multiplication, so 3 couldn't be on the right side.
                            $endgroup$
                            – no0ob
                            Jan 2 at 23:34






                            $begingroup$
                            for the order problem you could remove three zeros from the Least significant digit side and say it still needs to be divisible by 2 or 4 cause there are lots of 2's and 4's in the original multiplication, so 3 couldn't be on the right side.
                            $endgroup$
                            – no0ob
                            Jan 2 at 23:34














                            $begingroup$
                            The order is also given by the divisibilty rule for 7.
                            $endgroup$
                            – Darkice
                            Jan 3 at 8:54




                            $begingroup$
                            The order is also given by the divisibilty rule for 7.
                            $endgroup$
                            – Darkice
                            Jan 3 at 8:54











                            1












                            $begingroup$

                            Okay, $15! = 1*2*3..... *15=1a0767436b000$.



                            Why does it end with $000$? Well, obviously because $5,10,15$ all divide into it so $5^3$ divides into it and at least three copies of $2$ divid into it so $2^3*5^3 =1000$ divide into it.



                            If we divide $15!$ by $100 = 8*5^3$ we get



                            $1a0767436b = 1*2*3*4*6*7*9*2*11*12*13*14*3$



                            If we want to find the last digit of that we can do



                            $1a0767436b equiv b pmod {10}$ and



                            $1*2*3*4*6*7*9*2*11*12*13*14*3equiv 2*3*4*(-4)*(-3)*(-1)*2*1*2*3*4*3equiv$



                            $-2^9*3^4 equiv -512*81equiv -2 equiv 8pmod {10}$..



                            So $b = 8$.



                            But what is $a$?



                            Well, $11|1a0767436b$ and $9|1a0767436b$.



                            So $1+0+6+4+6 - a - 7-7-3-b = 11k$ for some integer $k$. And $1+a+0+7+6+7+4+3+6+b = 9j$ for some integer $j$.



                            So $-a -8 =11k$ so as $0le a le 9$ we have $a = 3$.



                            And that's that $15! = 1307674368000$..... IF we assume the person who asked this question was telling the truth.



                            We do know that $15!$ ends with $.... 8000$ but we are completely taking someone elses word for it that it begins with $1a0767436....$






                            share|cite|improve this answer









                            $endgroup$


















                              1












                              $begingroup$

                              Okay, $15! = 1*2*3..... *15=1a0767436b000$.



                              Why does it end with $000$? Well, obviously because $5,10,15$ all divide into it so $5^3$ divides into it and at least three copies of $2$ divid into it so $2^3*5^3 =1000$ divide into it.



                              If we divide $15!$ by $100 = 8*5^3$ we get



                              $1a0767436b = 1*2*3*4*6*7*9*2*11*12*13*14*3$



                              If we want to find the last digit of that we can do



                              $1a0767436b equiv b pmod {10}$ and



                              $1*2*3*4*6*7*9*2*11*12*13*14*3equiv 2*3*4*(-4)*(-3)*(-1)*2*1*2*3*4*3equiv$



                              $-2^9*3^4 equiv -512*81equiv -2 equiv 8pmod {10}$..



                              So $b = 8$.



                              But what is $a$?



                              Well, $11|1a0767436b$ and $9|1a0767436b$.



                              So $1+0+6+4+6 - a - 7-7-3-b = 11k$ for some integer $k$. And $1+a+0+7+6+7+4+3+6+b = 9j$ for some integer $j$.



                              So $-a -8 =11k$ so as $0le a le 9$ we have $a = 3$.



                              And that's that $15! = 1307674368000$..... IF we assume the person who asked this question was telling the truth.



                              We do know that $15!$ ends with $.... 8000$ but we are completely taking someone elses word for it that it begins with $1a0767436....$






                              share|cite|improve this answer









                              $endgroup$
















                                1












                                1








                                1





                                $begingroup$

                                Okay, $15! = 1*2*3..... *15=1a0767436b000$.



                                Why does it end with $000$? Well, obviously because $5,10,15$ all divide into it so $5^3$ divides into it and at least three copies of $2$ divid into it so $2^3*5^3 =1000$ divide into it.



                                If we divide $15!$ by $100 = 8*5^3$ we get



                                $1a0767436b = 1*2*3*4*6*7*9*2*11*12*13*14*3$



                                If we want to find the last digit of that we can do



                                $1a0767436b equiv b pmod {10}$ and



                                $1*2*3*4*6*7*9*2*11*12*13*14*3equiv 2*3*4*(-4)*(-3)*(-1)*2*1*2*3*4*3equiv$



                                $-2^9*3^4 equiv -512*81equiv -2 equiv 8pmod {10}$..



                                So $b = 8$.



                                But what is $a$?



                                Well, $11|1a0767436b$ and $9|1a0767436b$.



                                So $1+0+6+4+6 - a - 7-7-3-b = 11k$ for some integer $k$. And $1+a+0+7+6+7+4+3+6+b = 9j$ for some integer $j$.



                                So $-a -8 =11k$ so as $0le a le 9$ we have $a = 3$.



                                And that's that $15! = 1307674368000$..... IF we assume the person who asked this question was telling the truth.



                                We do know that $15!$ ends with $.... 8000$ but we are completely taking someone elses word for it that it begins with $1a0767436....$






                                share|cite|improve this answer









                                $endgroup$



                                Okay, $15! = 1*2*3..... *15=1a0767436b000$.



                                Why does it end with $000$? Well, obviously because $5,10,15$ all divide into it so $5^3$ divides into it and at least three copies of $2$ divid into it so $2^3*5^3 =1000$ divide into it.



                                If we divide $15!$ by $100 = 8*5^3$ we get



                                $1a0767436b = 1*2*3*4*6*7*9*2*11*12*13*14*3$



                                If we want to find the last digit of that we can do



                                $1a0767436b equiv b pmod {10}$ and



                                $1*2*3*4*6*7*9*2*11*12*13*14*3equiv 2*3*4*(-4)*(-3)*(-1)*2*1*2*3*4*3equiv$



                                $-2^9*3^4 equiv -512*81equiv -2 equiv 8pmod {10}$..



                                So $b = 8$.



                                But what is $a$?



                                Well, $11|1a0767436b$ and $9|1a0767436b$.



                                So $1+0+6+4+6 - a - 7-7-3-b = 11k$ for some integer $k$. And $1+a+0+7+6+7+4+3+6+b = 9j$ for some integer $j$.



                                So $-a -8 =11k$ so as $0le a le 9$ we have $a = 3$.



                                And that's that $15! = 1307674368000$..... IF we assume the person who asked this question was telling the truth.



                                We do know that $15!$ ends with $.... 8000$ but we are completely taking someone elses word for it that it begins with $1a0767436....$







                                share|cite|improve this answer












                                share|cite|improve this answer



                                share|cite|improve this answer










                                answered Jan 3 at 0:53









                                fleabloodfleablood

                                71.7k22686




                                71.7k22686























                                    1












                                    $begingroup$

                                    Let $d_1$ and $d_2$ be the two unknown digits.



                                    The number must be divisible by $8000$, because $15!$ contains $8$ and $1000$.



                                    $d_2$ is non-zero number, because $15!$ contains only three $5$s. It implies $1d_10767436d_2$ must be divisible by $8$. It implies $36d_2$ is divisible by $8$. Hence, $d_2=8$.



                                    Now you can use the divisibility by $9$ ($d_1+d_2=11$) and find $d_1=3$.






                                    share|cite|improve this answer









                                    $endgroup$


















                                      1












                                      $begingroup$

                                      Let $d_1$ and $d_2$ be the two unknown digits.



                                      The number must be divisible by $8000$, because $15!$ contains $8$ and $1000$.



                                      $d_2$ is non-zero number, because $15!$ contains only three $5$s. It implies $1d_10767436d_2$ must be divisible by $8$. It implies $36d_2$ is divisible by $8$. Hence, $d_2=8$.



                                      Now you can use the divisibility by $9$ ($d_1+d_2=11$) and find $d_1=3$.






                                      share|cite|improve this answer









                                      $endgroup$
















                                        1












                                        1








                                        1





                                        $begingroup$

                                        Let $d_1$ and $d_2$ be the two unknown digits.



                                        The number must be divisible by $8000$, because $15!$ contains $8$ and $1000$.



                                        $d_2$ is non-zero number, because $15!$ contains only three $5$s. It implies $1d_10767436d_2$ must be divisible by $8$. It implies $36d_2$ is divisible by $8$. Hence, $d_2=8$.



                                        Now you can use the divisibility by $9$ ($d_1+d_2=11$) and find $d_1=3$.






                                        share|cite|improve this answer









                                        $endgroup$



                                        Let $d_1$ and $d_2$ be the two unknown digits.



                                        The number must be divisible by $8000$, because $15!$ contains $8$ and $1000$.



                                        $d_2$ is non-zero number, because $15!$ contains only three $5$s. It implies $1d_10767436d_2$ must be divisible by $8$. It implies $36d_2$ is divisible by $8$. Hence, $d_2=8$.



                                        Now you can use the divisibility by $9$ ($d_1+d_2=11$) and find $d_1=3$.







                                        share|cite|improve this answer












                                        share|cite|improve this answer



                                        share|cite|improve this answer










                                        answered Jan 3 at 4:55









                                        farruhotafarruhota

                                        20.6k2740




                                        20.6k2740























                                            1












                                            $begingroup$

                                            $15!=2^{11}cdot 5^3cdot 7^2cdot 11cdot 13=(1000)X$ where $X=2^8cdot 3^6cdot 7^2cdot 11cdot 13.$



                                            The last digits of $2^8(=16^2), 3^6 (=9^3),7^2, 11,13$ are, respectively $6,9,9,1,3 .$



                                            Modulo $10$ we have $6cdot 9cdot 9 cdot 1cdot 3equiv 6cdot(-1)^2cdot 3equiv 18equiv 8$. So the last digit of $X$ is an $8$.



                                            Therefore the 2nd digit of $15!$ must be a $3$ in order for the sum of all its digits to be divisible by $9$.






                                            share|cite|improve this answer









                                            $endgroup$


















                                              1












                                              $begingroup$

                                              $15!=2^{11}cdot 5^3cdot 7^2cdot 11cdot 13=(1000)X$ where $X=2^8cdot 3^6cdot 7^2cdot 11cdot 13.$



                                              The last digits of $2^8(=16^2), 3^6 (=9^3),7^2, 11,13$ are, respectively $6,9,9,1,3 .$



                                              Modulo $10$ we have $6cdot 9cdot 9 cdot 1cdot 3equiv 6cdot(-1)^2cdot 3equiv 18equiv 8$. So the last digit of $X$ is an $8$.



                                              Therefore the 2nd digit of $15!$ must be a $3$ in order for the sum of all its digits to be divisible by $9$.






                                              share|cite|improve this answer









                                              $endgroup$
















                                                1












                                                1








                                                1





                                                $begingroup$

                                                $15!=2^{11}cdot 5^3cdot 7^2cdot 11cdot 13=(1000)X$ where $X=2^8cdot 3^6cdot 7^2cdot 11cdot 13.$



                                                The last digits of $2^8(=16^2), 3^6 (=9^3),7^2, 11,13$ are, respectively $6,9,9,1,3 .$



                                                Modulo $10$ we have $6cdot 9cdot 9 cdot 1cdot 3equiv 6cdot(-1)^2cdot 3equiv 18equiv 8$. So the last digit of $X$ is an $8$.



                                                Therefore the 2nd digit of $15!$ must be a $3$ in order for the sum of all its digits to be divisible by $9$.






                                                share|cite|improve this answer









                                                $endgroup$



                                                $15!=2^{11}cdot 5^3cdot 7^2cdot 11cdot 13=(1000)X$ where $X=2^8cdot 3^6cdot 7^2cdot 11cdot 13.$



                                                The last digits of $2^8(=16^2), 3^6 (=9^3),7^2, 11,13$ are, respectively $6,9,9,1,3 .$



                                                Modulo $10$ we have $6cdot 9cdot 9 cdot 1cdot 3equiv 6cdot(-1)^2cdot 3equiv 18equiv 8$. So the last digit of $X$ is an $8$.



                                                Therefore the 2nd digit of $15!$ must be a $3$ in order for the sum of all its digits to be divisible by $9$.







                                                share|cite|improve this answer












                                                share|cite|improve this answer



                                                share|cite|improve this answer










                                                answered Jan 3 at 10:16









                                                DanielWainfleetDanielWainfleet

                                                35.3k31648




                                                35.3k31648






























                                                    draft saved

                                                    draft discarded




















































                                                    Thanks for contributing an answer to Mathematics Stack Exchange!


                                                    • Please be sure to answer the question. Provide details and share your research!

                                                    But avoid



                                                    • Asking for help, clarification, or responding to other answers.

                                                    • Making statements based on opinion; back them up with references or personal experience.


                                                    Use MathJax to format equations. MathJax reference.


                                                    To learn more, see our tips on writing great answers.




                                                    draft saved


                                                    draft discarded














                                                    StackExchange.ready(
                                                    function () {
                                                    StackExchange.openid.initPostLogin('.new-post-login', 'https%3a%2f%2fmath.stackexchange.com%2fquestions%2f3059994%2fdetermining-the-missing-digits-of-15-equiv-1-square0767436-square000-without%23new-answer', 'question_page');
                                                    }
                                                    );

                                                    Post as a guest















                                                    Required, but never shown





















































                                                    Required, but never shown














                                                    Required, but never shown












                                                    Required, but never shown







                                                    Required, but never shown

































                                                    Required, but never shown














                                                    Required, but never shown












                                                    Required, but never shown







                                                    Required, but never shown







                                                    Popular posts from this blog

                                                    Quarter-circle Tiles

                                                    build a pushdown automaton that recognizes the reverse language of a given pushdown automaton?

                                                    Mont Emei